11
$\begingroup$

This question is partly inspired by a problem in Stewart's Calculus: "Find the area of the region enclosed by $y=x^2$ and $x=y^2$."

Suppose $f\colon [0,1]\to [0,1]$ is a convex increasing function that fixes 0 and 1. The graphs $y=f(x)$ and $x=f(y)$ enclose a convex region. To find its area, students will likely integrate the length of vertical sections: $f^{-1}(x)-f(x)$. They are unlikely to ask if two different functions can yield the same integrand. But I'll ask:

Does $f^{-1}-f$ determine $f$? (Among all convex increasing functions that fix 0 and 1).

$\endgroup$
1
  • 1
    $\begingroup$ It would be better to write "strictly increasing" in place of "increasing", to guarantee that $f^{-1}$ exists. $\endgroup$ Jan 10, 2010 at 19:45

1 Answer 1

9
$\begingroup$

Yes.

Assume we have two distinct functions $f$ and $g$ such that $f^{-1}-f\equiv g^{-1}-g$. Take a sequence $x_n=f(x_{n-1})$. Clearly $f(x_n)-g(x_n)=0$ or $(-1)^n[f(x_n)-g(x_n)]$ has the same sign for all $n$.

Sinse $\int_0^1f=\int_0^1g$, there are two sequences $x_n$ and $y_n$ as above such that $f(x_n)=g(x_n)$, $f(y_n)=g(y_n)$ and say $(-1)^n[f(x)-g(x)]>0$ for any $x\in(x_n,y_n)$. Note that $x_n,y_n\to 0$ and $\int_{x_n}^{y_n}|f-g|=const>0$. It follows that $\limsup_{x\to0} |f(x)-g(x)|\to\infty$, a contradiction.

$\endgroup$
5
  • 1
    $\begingroup$ Great. It took me some thought to figure out why the integral is constant towards the end. In case others have the same difficulty: Write the integral as $\pm\int_{x_n}^{y_n}(f^{-1}(y)-g^{-1}(y))\,dy$ and deal with the integral of each addend separately. Substitute $y=f(x)$ in the first integral and perform a partial integration to deal with the resulting factor $x$. Ditto in the second integral, with $y=g(x)$. Subtract. $\endgroup$ Jan 10, 2010 at 14:08
  • 3
    $\begingroup$ I personally feel that almost every sentence in this solution could be improved by including the justifications for claims made, starting with the sentence beginning "Clearly...". I hope that Anton will choose to expand it! $\endgroup$ Jan 10, 2010 at 19:43
  • 2
    $\begingroup$ @Bjorn, I personally do not like to read proofs which contains ALL details :) $\endgroup$ Jan 11, 2010 at 0:07
  • $\begingroup$ Well, in any case, it's a nice solution. $\endgroup$ Jan 11, 2010 at 6:24
  • $\begingroup$ This is one candidate for a proof by picture. The three "confusing" sentences starting with "Clearly", "Sinse" (sic), and "Note" are amazingly clear once you draw a picture similar to what Leonid describes. $\endgroup$ Apr 22, 2010 at 10:56

Your Answer

By clicking “Post Your Answer”, you agree to our terms of service and acknowledge that you have read and understand our privacy policy and code of conduct.

Not the answer you're looking for? Browse other questions tagged or ask your own question.